except

This topic has expert replies
Master | Next Rank: 500 Posts
Posts: 132
Joined: Thu Dec 02, 2010 2:49 am
Thanked: 5 times

by RadiumBall » Sun Feb 27, 2011 10:52 pm
I agree that A claims the number of smokers have increased but more number of smokers would obviously mean higher tobacco sales very much against the question stem...

So if A is correct then is the other part of the question stem i.e. tobacco sales not important? How do we identify such situation?

Legendary Member
Posts: 1574
Joined: Fri Jan 28, 2011 2:52 am
Thanked: 88 times
Followed by:13 members

by aspirant2011 » Mon Feb 28, 2011 3:03 am
I would also go with A option because if we have 8 women who have started and 5 men who left then 3 adults have got increased.............. we were supposed to find an answer choice in which tobacco consumption has increased but number of adults have also got increased..........

B talks about teen age children therefore wrong.......
C doesnt mention anything about adults, it just talks about non smokers and moreover question asks us for "decrease in the number of adults"...........
D irrelevant as it doesnt talk anything about the number of people who have started smoking
E same as D..........

Newbie | Next Rank: 10 Posts
Posts: 7
Joined: Wed Feb 09, 2011 8:04 pm
Thanked: 3 times

by Joe K » Mon Feb 28, 2011 11:33 am
I get what you are saying Clawhammer, and D could fall victim to the same type of analysis. I think it needs to be looked at like this though:

Does B explain how both facts COULD be true? Yes.
Does A explain how both facts COULD be true? No.

Is B a perfect, lock-tight, has to be the explanation, answer? No, but it could be one, and that's what the question stem asks for.
A does nothing to address the paradox, if anything it adds to it.

So you are right in the way you showed how under certain scenarios B might not actually address the paradox, but under certain scenarios it does. A does not under any circumstance.

Master | Next Rank: 500 Posts
Posts: 109
Joined: Mon Jan 10, 2011 3:05 am
Thanked: 8 times
Followed by:7 members
GMAT Score:760

by nikhilsrl » Tue Mar 01, 2011 2:11 am
A very good question. At first glance all choices seem to explain the paradox. But A is tricky - it says that the number of women who have taken up smoking is more than the number of men who stopped, but does not consider women who may have stopped smoking. The stimulus mentions that number of adults (both men and women) have decreased. So it is A.

User avatar
Legendary Member
Posts: 979
Joined: Tue Apr 14, 2009 1:38 am
Location: Hyderabad, India
Thanked: 49 times
Followed by:12 members
GMAT Score:700

by bubbliiiiiiii » Wed Mar 02, 2011 12:09 am
IMO A
Regards,

Pranay

Junior | Next Rank: 30 Posts
Posts: 16
Joined: Fri Jun 24, 2011 6:46 am

by trangle.nh » Thu Jul 07, 2011 8:00 pm
Let look of the answer as follows:
Paradox here:
P1: number of adult smokers decreased (demand from one source)
P2: total amount of tobacoo sold increased (supply)

Prediction: demand still increased but from the other sources of demand
Reasoning: there are many sources of demand affecting the supply of one product. So decrease in one source may not lead to the decrease in overall supply since the overall supply is decided by the overall demand.

solution:
A. During this year, the number of women who have begun to smoke is greater than the number of men who have quit smoking.
--> Only mention demand of adults which totally decreased anyway--> Not solve the paradox

B. The number of teen-age children who have begun to smoke this year is greater than the number of adults who have quit smoking during the same period.
--> Mention the increase new demand of teen-age children, which exceed the decrease in demand of adults --> This may solve the paradox (since it assumed that: others not mentioned are unchanged)
C. During this year, the number of nonsmokers who have begun to use chewing tobacco or snuff is greater than the number of people who have quit smoking.
--> Same as B: it may solve the paradox
D. The people who have continued to smoke consume more tobacco per person than they did in the past.
--> Same as B&C: it may solve the paradox (since it did not take into account the number of people chewing the tobbaco)
E. More of the cigarettes made in the United States this year were exported to other countries than was the case last year.[/quote]
--> It may solve the paradox again

To solve this type of Q, we need to assume that: others not mentioned are unchanged

So B,C,D,E may solve the paradox but A did not --> A is correct

P/S: I have a small problem: Is women + men -> adults?

User avatar
Master | Next Rank: 500 Posts
Posts: 461
Joined: Tue May 10, 2011 9:09 am
Location: pune
Thanked: 36 times
Followed by:3 members

by amit2k9 » Fri Jul 08, 2011 12:28 am
classic usage of proportion in A here.
A is clear and clean.
For Understanding Sustainability,Green Businesses and Social Entrepreneurship visit -https://aamthoughts.blocked/
(Featured Best Green Site Worldwide-https://bloggers.com/green/popular/page2)

Master | Next Rank: 500 Posts
Posts: 135
Joined: Thu May 05, 2011 9:00 am
Thanked: 4 times
Followed by:1 members
GMAT Score:700

by mirantdon » Sat Jul 09, 2011 10:31 am
+1 for A .

Newbie | Next Rank: 10 Posts
Posts: 9
Joined: Mon Jul 04, 2011 7:12 am
Thanked: 1 times

by nguy » Sat Jul 09, 2011 11:37 pm
One more A

User avatar
Senior | Next Rank: 100 Posts
Posts: 52
Joined: Sun Mar 13, 2011 12:07 am
Location: Pune, India
GMAT Score:680

by DhruvXVII » Sun Jul 10, 2011 12:09 am
Clearly, A does not help in explaining the paradox.

User avatar
Master | Next Rank: 500 Posts
Posts: 123
Joined: Tue May 31, 2011 12:26 am
Location: Hyderabad
Thanked: 5 times
Followed by:1 members

by jainnikhil02 » Sun Jul 10, 2011 12:32 am
IMO A
Nikhil K Jain
____________________

"Life is all about timing" Don't waste your and others time.